Which one of the following, if substituted for the restriction that if music is taken, then neither physics nor theat...

Amaris on June 14, 2015

Please explain

How would we go about answering this question?

Replies
Create a free account to read and take part in forum discussions.

Already have an account? log in

Melody on June 17, 2015

You can access the full video explanation for this game by tapping on the "play" button.

Hope that clears things up! Please let us know if you have any other questions.

Richa on October 2, 2020

I still don't understand even after the video

Shartecia on October 29, 2020

Me either

Victoria on November 13, 2020

Hi @RS1 and @tisha212@yahoo.com,

Happy to help!

We know that a student is choosing courses to take during a summer school session.

Each student in summer school must take at least 3 courses from the following seven: H, L, M, P, S, T, and W.

Let's go through our conditions.

Rule 1 - If H is taken, then neither S nor M can be taken

H --> Not S and Not M
S or M --> Not H

This means that H can only be taken with L, P, T, and/or W.

Rule 2 - If M is taken, then neither P nor T can be taken.

M --> Not P and Not T
P or T --> Not M

This means that M can only be taken with H, L, S, and/or W.

However, we know from Rule 1 that, if M is taken, then H cannot be taken. Therefore, M can only be taken with L, S, and/or W.

Rule 3 - If W is taken, then neither S nor P can be taken.

W --> Not P and Not S
P or S --> Not W

This means that W can only be taken with H, L, M, and/or T.

We are looking for the answer choice which, if substituted for Rule 2, would have the same effect in determining which courses the student can take.

Let's go back to the deduction we made above. If M is taken, then neither P nor T can be taken.

Therefore, if the student takes M, they could also take H, L, S, and/or W.

However, Rule 1 tells us that if the student takes H, they cannot also take M.

Therefore, if the student takes M, they can only also take L, S, and/or W.

Notice that this deduction is directly restated by answer choice (B), making it our correct answer.

Answer choice (A) is incorrect because it is possible that P or T are taken with M as we have swapped out Rule 2 for this new rule.

Answer choice (C) is incorrect because it is too narrow. It only focuses on P, while leaving it possible that M and T can be taken together.

Answer choice (D) is incorrect for the same reason as answer choice (C); however, it is focused on T instead.

Answer choice (E) is incorrect because it overlooks the possibility that M could be taken with only one of P or T.

Hope this helps! Please let us know if you have any further questions.